LSAT and Law School Admissions Forum

Get expert LSAT preparation and law school admissions advice from PowerScore Test Preparation.

 Administrator
PowerScore Staff
  • PowerScore Staff
  • Posts: 8916
  • Joined: Feb 02, 2011
|
#84492
Complete Question Explanation

The correct answer choice is (E).

Answer choice (A):

Answer choice (B):

Answer choice (C):

Answer choice (D):

Answer choice (E): This is the correct answer choice.

This explanation is still in progress. Please post any questions below!
 lolaSur
  • Posts: 46
  • Joined: Nov 11, 2019
|
#73463
Could you explain D and E please?

I originally chose C. I went back and did the question again and realized that C isn't correct because there aren't any "divergent" opinions. Is answer C considered an extreme answer?

answer D - although a critic's point is expressed in paragraph 2: "some critics point to the patronizing..." I didn't think the author was necessarily assessing claims because in paragraph 3 and 4 the author expands, through examples, the point that Native Americans have benefited from exercise of judicial power. Am I missing anything?

Answer E - The contention supported is that Native Americans have benefitted from the exercise of judicial power. Please confirm if this is correct.

I am not very clear on what the role of the word accommodating is here. My instinct originally was that "accommodating"
is a positive thing, but the word "despite" makes "accommodating" sound like it is not a positive thing. Is not really clear to me what "accommodating" means in this passage.

Thank you!

For my reference: (PT 1995, RC, q14 law diversity)
 Claire Horan
PowerScore Staff
  • PowerScore Staff
  • Posts: 408
  • Joined: Apr 18, 2016
|
#73488
Hi LolaSur,

The best way to approach this problem is to prephrase your own statement of the author's primary purpose before looking at the answer choices. If you did this, E would be more likely to stand out as the right answer.

To answer your questions, here's what's wrong with C and D.

(C) describes a hypothetical passage where an author presents two opinions that seem to be in conflict, but the author synthesizes them into his/her own conclusion. It's not really "extreme" in any sense. But, as you have already realized, that's not what's happening in the passage.

A passage that did what (D) describes would evaluate two or more contentions or theories that were in conflict with each other. What are those contentions and who are they attributed to? Does the author really evaluate which of several claims are correct? That doesn't really seem like the passage.

(E) describes a passage that has a thesis and supports it with evidence. That's what we have here. The author's thesis is that the Supreme Court's authority has generally been somewhat beneficial or not completely detrimental the Native Americans. The author's evidence consists of (1) the rules of construction the Supreme Court has created and (2) the protection the Supreme Court has offered from states' encroachment into tribal affairs.

When weighing the answer choices in a question like this, pay attention to the structure of the passage. This passage first makes an assertion, then supports it with two main points. Answer choice (E) is the closest to that structure.

Lastly, your question about the word "accommodating." The fact that the author puts the words in quotation marks suggests he or she is somewhat skeptical, so even though it is usually a positive word, it is not used in a positive way here. To figure out what "accommodating" means in the passage requires you to look at how it's used in context (not simply by checking your own vocabulary). If you look in the passage before "accommodating" was used, you'll see the author's discussion of compromises to the political realities of the time. The Supreme Court was "accommodating" the views of the majority, politicians, states, etc.

I hope this explanation helps!

Get the most out of your LSAT Prep Plus subscription.

Analyze and track your performance with our Testing and Analytics Package.